Zero-sum subset problem

NightBreeze

New member
Joined
Jun 22, 2008
Messages
4
Hello everyone.

I have been working on a math problem for some time now, but I can not finish the proof. In other words, I'm stuck. I hope someone here can solve it for me, or perhaps we could make it into a joint effort. The problem is taken from a monthly math magazine, which features 3 math problems each month.

Here's the problem:

---------------------------------------------------------------------------------------
Consider a set \(\displaystyle S\subset\mathbb{Z}\) with \(\displaystyle |S| = 15\) and the following property:

\(\displaystyle \forall s\in S\), \(\displaystyle \exists a,b\in S\) such that \(\displaystyle s = a + b\).

Proof that for every such \(\displaystyle S\), there exists a non-empty subset \(\displaystyle T\subset S\) with \(\displaystyle |T|\leq 7\) of which the elements sum up to zero.
---------------------------------------------------------------------------------------


I have come up with a few observations that might be useful, but I think it's best to wait and see how things go. If noone has an idea how to approach the problem or if someone asks me for the info, then I will give it.

Until then, good luck!
 
Just a quick obervation, I don't have much time to think about it. Let b1 through b15 be the elements of S. Assume S is sorted from least to greatest.

Since b1 is the smallest element we must have b1=b1+0 be the only two elements satisfying the property governing S. So 0 is in S.

Also a question: the problem as stated doesn't require this, but are a and b necessarily different, or can they be the same?
 
daon said:
Since b1 is the smallest element we must have b1=b1+0 be the only two elements satisfying the property governing S. So 0 is in S.
That is not the case. I think that you have misread or misinterpreted the question.
You have assumed that the domain is the set of non-negative integers.
But S could contain {-5,-3,-2,2,3,…}. Note that the sum of any two elements of S need not be in S.
 
The proof is rather trivial if S contains zero or if some s in S has its inverse also in S. We may rightly assume S contains neither.

The problem may seem innocent, but there are quite a few pitfalls along the way to the proof..

It's good to see people thinking about it though : )
 
NightBreeze said:
I hope someone here can solve it for me....
I'm sorry, but tutors help the students do the work on their own. Just doing the work for you wouldn't help you learn (and is in any case often viewed by contest adjudicators, instructors, etc, as "cheating"). :shock:

NightBreeze said:
I have come up with a few observations that might be useful, but I think it's best to wait and see how things go.... The problem may seem innocent, but there are quite a few pitfalls along the way to the proof....
Um... Actually, the only way the tutors can know what you've done and what you're thinking is, I'm afraid, for you to actually tell us. Sorry! :oops:

Please reply with a complete listing of all of your work and reasoning so far, clearly stating where you are having difficulty. Thank you! :D

Eliz.
 
This is not a homework assignment nor a problem which I am working on with the intention of learning something. I realize that this goes against the general philosophy of this forum as a tutoring community but, in all honesty, I posted it because I can't for the life of me figure out how to proof it. Secondly, my reasons for wanting to know the solution have nothing to do with contests or anything else in which asking for help would be cheating. I am genuinly interested in the proof.

That being said, I suppose I can describe what I have done so far. I was postponing because I did not want people to start working on the problem with some sort of a 'tunnel vision' on which way to think about it, hoping to get some fresh ideas.

So, some observations so far:
  • - We may assume that S contains neither 0 nor an inverse of another member of S, for this would make choosing T fairly trivial. (i.e.: T = {0} or T = {s, -s})

    - With this assumption, it can be proven that S has to contain at least three positive and three negative numbers. This implies that the number of negative numbers in S is between 3 and 7, limits including. (The proof is simply a considerable number of logical steps. Not really interesting, but I can share it with you if required).

    - For |S| = 2n with n in the natural numbers, it can be shown that there exists an S so that the smallest subset T which sums to zero contains n elements. This means that the upper limit of 7 in the case of 15 can not be decreased and is not valid in the case of |S| = 16. The proof is rather tedious, but in the case of |S| = 16, consider the following set as a counter example to lowering the upper bound on the cardinality of T below 8:
    S ={–254, –253, –251, –247, –239, –223, –191, –127, 1, 2, 4, 8, 16, 32, 64, 128}.
    You can see from the pattern that this can be extended to any even size. For cardinality of the form 2n+1, simply construct a counter example as above for the case |S| = 2n and add double the smallest negative number.

    - The smallest positive and the largest negative number are both written as the sum of both a positive and a negative number.

The most obvious approach would be to somehow choose a member s of S in such a way that by 'expanding' s as the sum of other member of S, the same element will be encountered implying that the sum of the remaining members in the sum add up to zero. The problem with this, however, is that not all members of S occur in a subset T which sums to zero and it can not be guaranteed that there will be no doubles in the 'expansion' of a member s. However, it might be possible to choose s in such a way that this will work. I don't know how to go about it though.

It seems some more properties of the internal structure of such an S have to be found before we can proceed. Perhaps make a graph out of it?

I have also considered some different notation. Let \(\displaystyle s = (s_1, s_2, \ldots, s_n)\) be a list of the elements of S. Then for each i, there exists f(i) and g(i) such that \(\displaystyle s_i = s_{f(i)} + s_{g(i)}\). Then there is an n x n matrix A with As = 0, namely
\(\displaystyle A = I - \sum_{i=1}^n (E_{i,f(i)} + E_{i,g(i)})\)
, where \(\displaystyle E_{i,j}\) is the matrix with 1 in the position (i,j) and 0 elsewhere. This notation changes the problem into finding a vector in the row span of A with a certain property.


I do not know how to proceed and need some new ideas, I'm afraid. The problem seems as innocent as a simple textbook problem to introduce some new ideas, but it's actually trickier than apparant at first sight.
 
I can update my thoughts so far, perhaps someone will be interested enough to take a look at it.

--------------
Let S be a set of integers with |S| = 15. We may assume that S contains neither zero nor an inverse of another member of s, for it would make the choice for our subset T rather trivial. Furthermore, if S contains neither, it can be shown that it must contain at least three positive and three negative numbers. The proof is rather simple, just some logic steps. So take my word for it for now.
Consider now A to be the positive members of S and B to be the negative members of S. Without loss of generality, we may assume that |A| < |B|. So 3 <= |A| <= 7. Now define a graph G on A as follows: Let x, y be members of A. There exists an edge x -> y if there exists a z in S such that x = y + z. By this definition, every member of A is adjacent to at least one other member of A, because every positive number can not be written as the sum of two negative numbers. Because A is finite, there must exist a cycle X = x1, ... xk. Let k be the smallest k with this property. Then there exist Z = z1, ... zk such that

x1 = x2 + z1 = x3 + z2 + z1 = ... = x1 + zk + ... + z1.

The the sequence z1, ..., zk will sum to zero.

The way I see it, the main problem in this proof has to do with the doubles that occur in such a sequence Z. If there are no doubles in the sequence, then we have found our T. So let's assume that there are x[sub:2yewf1qi]i[/sub:2yewf1qi] and xj with i =/= j such that xi = xi+1 + z and xj = xj+1 + z. It is easy to show that z can not be a member of X. If it were, then there would exist a smaller cycle through xi, xj and z. That is not allowed since we chose X to be the smallest cycle in A. So we are left with the case that z is not a member of X. We need to proof that either we can replace z and still get a T with |T| <= 7 or that there must exist a different cycle (possibly in B) with the required properties.

I experimented some with assuming that |X| < 7. Then it's possible for z to be positive. There are some implications there, but nothing useful. We can 'expand' z a number of times till we reach a member of X. A new cycle might be possible in that way. Remember that our upper limit on the cycle size is equal to the upper limit on the amount of positive numbers in S. The other possibility is that z is in B. Maybe it's worth looking into the smallest cycle in B then? We have no guarantee that its length will be smaller than 7, but maybe we can proof that the biggest cycle in B can be no larger than the size of A. This certainly sounds plausible.. since negative numbers are absolutely required to make a cycle in the positive part of S.
--------------

The set I've been looking into most recently is S = {-8, -4, -3, -1, 2, 5, 7, 9}. It's of size 8, naturally, but the cycle 9 -> 7 -> 5 -> 9 has a complement Z which features the number 2 twice. I'm trying to find a structural way to avoid this. My work so far:

-------------
Let X be the smallest cycle x1 -> ... -> x1 in A (the positive members of S). Then 3 <= |X| <= 7, because 3 <= |A| <= 7 and |X| = 1 -> 0 in S, |X| = 2 -> S has an inverse.

Now let xi and xj be such that xi = xi+1 + z and xj = xj+1 + z and suppose that i < j. Now there are a few cases I was thinking of addressing.

- Suppose z = xk in X. Then there exists a cycle inside of X which is smaller than X itself. This is not possible since I chose X to be the smallest cycle in A.
- Suppose |X| < 7. Then there can possibly be m = |A| - |X| members of A which are not in X. So suppose that z in A \ X. If we expand z m times, we must arrive at a member of X (since there are only m members of A left which are not in X). Then we can construct a new cycle which includes z and might possibly be larger than X but must be smaller or equal than 7.

The final case is when z is in B. I don't know how to approach that one.

Does this make sense?


Edit: I reread it and concluded, once again, that graph theory proof are totally unreadable without some drawing or example. So I'll just give you one.

Consider S = {-8, -4, -3, -1, 2, 5, 7, 9}. Then |S| = 8 and believe me that it satisfies the important property. Then A = {2, 5, 7, 9} and we found the cycle 9 -> 7 -> 5 -> 9 with complement {2, 2, -4}. In this case, xi = 9, xj = 7 and z = 2. Note that |X| = 3 < 4 = |A|, so m = 1. Here, z is in amongst the remaining members of A which are not in X, so z in X \ A. Note that if we expand z, we get z = 5 + -3. 5 is a member of X, which is to be expected after m expansions. Now we can construct the cycle z -> 5 -> 9 -> z (because 9 = xi) which satisfies our criterium.

The trick here is that the length of such a newly found cycle can not exceed |A|, which is what we wanted.
-------

There is no guarantee that this new cycle will not conflict, but I'm fairly confident we can repeat this process till we find a cycle in A which either doesn't have a conflict, or the conflicting z is not a member of A.

Hope this helps...
 
Top